Elementary Linear Algebra 7th Edition

Published by Cengage Learning
ISBN 10: 1-13311-087-8
ISBN 13: 978-1-13311-087-3

Chapter 4 - Vector Spaces - 4.3 Subspaces of Vector Spaces - 4.3 Exercises - Page 167: 34

Answer

$W$ is not closed under addition and hence it is not a vector subspace of $M_{n,n}$.

Work Step by Step

The set $W$ of all $n \times n$ invertible matrices is not a vector subspace of $M_{n,n}$. Indeed; the sum of two invertible matrices not necessarily invertible. So $W$ is not closed under addition and hence it is not a vector subspace of $M_{n,n}$.
Update this answer!

You can help us out by revising, improving and updating this answer.

Update this answer

After you claim an answer you’ll have 24 hours to send in a draft. An editor will review the submission and either publish your submission or provide feedback.